Can someone please find x for me? Thank you!

Can Someone Please Find X For Me? Thank You!

Answers

Answer 1

The value of the missing angle x of the triangle is; x = 18.67°

How to find the missing angles?

The sum of angles on a straight line is 180 degrees. Thus, the supplementary angle to 130 degrees is;

180 - 130 = 50°

Now, the alternate angle to 80 degrees which will be located inside the triangle is 80°.

The sum of angles in a triangle is 180 degrees. Thus;

3x - 6 + 80 + 50 = 180

3x + 124 = 180

3x = 180 - 124

3x = 56

x = 56/3

x = 18.67°

Read more about missing angles at; https://brainly.com/question/28293784

#SPJ1


Related Questions

Carmela's gross pay is $784.25. Her deductions total is $191.17. What percent of her gross pay is take-home pay?​

Answers

The percent of Carmela's gross pay that is her take-home pay is 75%

How to calculate the percent of Carmela's gross pay that is her take-home pay?

Carmela's gross pay is $784.25

Her total deductions is $191.17

Therefore the percent of her gross pay can be calculated as follows

The first step is to subtract the deductions from the gross pay

784.25 - 191.17

= 593.08

The percent of the gross pay that is take-home pay is

593.08/784.24 × 100

= 0.75 × 100

= 75%

Hence 75% of her gross pay is take-home pay

Read more on gross pay here

https://brainly.com/question/2890599

#SPJ1

Select the linear function that describes the relationship between the domain and range in the table below.


x f(x)

−1 5

0 3

1 1

Answers

The linear function that describes the relationship is y = -2x + 3

How to determine the linear function

From the question, we have the following parameters that can be used in our computation:

x f(x)

−1 5

0 3

1 1

A linear function is represented as

y = mx + c

Where

c = y when x = 0

This means thar

c = 3

So, we have

y = mx + 3

From the table, we can see that

As x increases by 1, y decrease by 2

This means that

m = -2

So, we have

y = -2x + 3

Hence, the equation is y = -2x + 3

Read more about linear function at

https://brainly.com/question/2030026

#SPJ1

Lucia and Danna both recently purchased fitness watches. The watches count the number of steps they walk in a day. At lunchtime, Lucia has 5445 steps and Danna has 4995 steps. Lucia averages 800 steps per hour and Danna averages 900 steps per hour.

A. Write and solve a system of linear equations that represent the total number of steps each person takes.
B. Interpret the meaning of the solution in terms of the problem situation.

Answers

The system of linear equations is y = 800x + 5445, and y = 900x + 4995. The solution is 4.50. This states that the number of steps taken by Danna and Lucia will be equal after 4.50 hours.

What is system of linear equation?

The set of two or more linear equations containing the same variables is known as a system of linear equations in mathematics. In this case, linear equations are first-order equations, where the maximum power of the variable is 1. One, two, or three variables may be present in a linear equation.

Let us suppose the number of hours = x.

Let us suppose the total number of steps = y.

Given that, Lucia has 5445 steps and Lucia averages 800 steps per hour.

For Lucia the linear equation is:

y = 800x + 5445

For Danna, Danna has 4995 steps and Danna averages 900 steps per hour.

For Danna the linear equation is:

y = 900x + 4995

The system of linear equations is:

y = 800x + 5445 (equation 1)

y = 900x + 4995 (equation 2)

Substitute the value of y of equation 1 in equation 2:

800x + 5445 = 900x + 4995

5445 - 4995 = 900x - 800x

450 = 100x

x= 4.50

B. The solution of the linear equation is 4.50. This states that the number of steps taken by Danna and Lucia will be equal after 4.50 hours.

Learn more about system of linear equations here:

https://brainly.com/question/12895249

#SPJ1

since the mode is the most frequently occurring data value, . a. it can never be larger than the mean b. it is always larger than the median c. it is always larger than the mean d. more than one mode can exist

Answers

The distribution of the mean, median, and mode values will depend on how skewed the distribution is. As a result, mean or mode may be larger than or less than mode.

correct answer is D.

Different measures of the center in a set of numerical data include mean, median, and mode. They each attempt to condense a dataset into a single number that may be used to represent a "typical" data point.

The "mean" value is obtained by summing all the data points and dividing the result by the total number of data points.

The middle number, or median, is obtained by placing all data points in order and choosing the middle one (or, if there are two middle numbers, the mean of those two figures).

know more about mean here

https://brainly.com/question/30112112#

#SPJ4

Does the point (0, 0) satisfy the equation y = x?

Answers

Answer:

yes

Step-by-step explanation:

when y equals 0, x equals 0 also

y = x will produce a straight line passing through the point of origin (0,0)

(x,y) => (0,0)

Plug in the point into the equation, y=x.

We get,

0=0, which is true.

Thus, the point, (0,0), satisfies the equation, y=x.

What is the greatest common factor shared by 39 and 15

Answers

Answer:

3

Step-by-step explanation:

15 is divisible by 3, 3 × 5 = 15. Then of course 1 and 15 no other ones. 1,3,5,15

39 is divisible by 3, 3× 13 = 39, then 1 and 39. 1,3,13,39

The only common factors are 1 and 3, and since 3 is greater that is the greatest common factor.

Hope that helps! Let me know if you have any further questions.

The greatest common factor shared by numbers 39 and 15 is 3.

Given two numbers, 39 and 15.

The greatest common factor can be defined as the factor which is common to both 39 and 15 and is the greatest of all those common factors of 39 and 15.

First, list out the factors of 39 and 15.

Factors of 39 = 1, 3, 13, 39

Factors of 15 = 1, 3, 5, 15

The only factor which is common to both 39 and 15 is 3.

So, the greatest common factor of both is 3.

Learn more about Greatest Common Factors here :

https://brainly.com/question/29584814

#SPJ6

At the nearest wholesale store, 3 bags of cereal cost $21.75. Which equation represents the proportional relationship?

Answers

The answer is 3b=$21.75

Find the vector equation for the line of intersection of the planes x+4y-4z=-3 and x+z=5. r= < , ,0> +t <4, , >.

Answers

The vector equation for the line of intersection of the planes x+4y-4z=-3 and x + z = 5 is r = <5, -2, 0> + t <-1, 5/4, 1>

The equations of the planes are:

x + 4y - 4z = -3                    ..............(1)

and x + z = 5                      .............(2)

Subtract equation (2) from equation (1),

( x + 4y - 4z + 3) - (x + z - 5 ) = 0

4y - 5z + 8 = 0

we can rewrite is as,

4y = 5z - 8

y = 5/4 z - 8/4

y = 5/4 z - 2                  ...............(3)

Here, we don't have enough equations to uniquely solve for either y or z. So, we set z = t, where t can be any real number.

The variable t is the free parameter for the parametrization of the line.

Substitute z = t in equation (3) we get,

y = 5/4 t - 2        

From equation (2),

x = 5 - z

For z = t,

x = 5 - t

Thus, the parametrization of the line would be,

(x, y, z) = (5 - t, -2 + 5/4 t , t)

           = <5, -2, 0> + t<-1, 5/4, 1>

Learn more about the equation of the line here:

https://brainly.com/question/30360806

#SPJ4

1. If mA = 3x° and
m what is the value
of x?

Answers

Answer: I'm sorry, but the information provided is not enough to determine the value of x. The statement "mA = 3x°" does not provide enough context to understand what "mA" and "x" represent, and what the relationship between them is. Additionally, the statement "m" does not provide any information that can help determine the value of x. Can you please provide more information or context?

Step-by-step explanation:

Consider this series.
16+32/3 +64/9 +128/27 +...
Does the series converge or diverge?
Select answers from the drop-down menus to correctly complete the statements.
The series Choose... You know this because the series is
Choose....

Answers

The series converges to 16 * 3 = 48

What is a series?

A number series is a continues chain of number identical of non-identical depending on the logic behind the series. It can contain letters, number, variable, fraction, and numbers with power or exponents.

How to find the series coverage?

The series converges.

This series is in the form of 16 * (1 + 1/3 + 1/9 + 1/27 + ...)

The series 1 + 1/3 + 1/9 + 1/27 + ... is a geometric series with a common ratio of 1/3.

A geometric series converges if |r| < 1, where r is the common ratio. In this case, |1/3| < 1, so the series converges to lim (1/(1-1/3)) = 3.

So the series converges to 16 * 3 = 48

To know more about number series visit:

https://brainly.com/question/12474324

#SPJ1

Some people took part in a game. The frequency shows information about their scores.ScoreFrequency - 758 - 10811 - 15916 - 201521 - 351636 - 5013Estimate the mean.Give your answer rounded to 2 decimal places.

Answers

The estimated mean score is given as follows:

19.12.

How to obtain the mean of a data-set?

The mean of a data-set is obtained as the sum of all observations in the data-set divided by the number of observations.

For each score, we consider it to be the mean of each interval in the frequency table, hence the sum of all scores is given as follows:

4 x 19 + 9 x 14 + 13 x 15 + 18 x 28 + 28 x 10 + 43 x 13 = 1740.

The total number of scores is of:

19 + 14 + 15 + 20 + 10 + 13 = 91.

Hence the mean score is obtained as follows:

1740/91 = 19.12.

Missing Information

The frequency distribution for this problem is given as follows:

1-7:19

8-10:14

11-15:15

16-20:20

21-35:10

36-50:13

More can be learned about the mean of a data-set at https://brainly.com/question/1156334

#SPJ1

The ratio of boys to girls in Mandy’s mathematics class is 3 to 12.
This ratio is the same in Mandy’s science class, in which there are 20 girls. How many boys are there in Mandy’s science class? Show your work

Answers

Number of boys in Mandy’s science class is 5.

Now, According to the question:

The ratio of boys to girls in Mandy’s mathematics class is 3 to 12.

This ratio is the same in Mandy’s science class,

There are 20 girls.

To find the how many boys are there in Mandy’s science class?

Based on the given condition:

Let the ratio be 3x and 12x

Now, According to the statement:

12x = 20

x = 20/12

x = 5/3

So, Number of boys = 3 × 5/3 = 5

Learn more about Ratio at:

https://brainly.com/question/13419413

#SPJ4

Write two expressions that could be used to determine what 340 increased by 19% is​

Answers

The new value = 340 + (19% × 340)

The actual change = The new value - 340

"Information available from the question"

340 increased by 19%

To write in mathematically:

The value of the percentage increase = 19% × 340

The new value = 340 + The value of the percentage increase

The new value = 340 + The value of the percentage increase

= 340 + (19% × 340)

= 340 + 19% × 340

= (1 + 19%) × 340

= (100% + 19%) × 340

= 119% × 340

= 119 ÷ 100 × 340

= 119 × 340 ÷ 100

= 40,460 ÷ 100

= 404.6

The actual change = The new value - 340

                              = 404.6 - 340

                              =  64.6

Learn more about Percentage at;

https://brainly.com/question/26097851

#SPJ4

Identify the number of terms and then the resend themselves for each algebraic expression

Answers

The number of terms in the algebraic expressions are:

6y + 14 ⇒ 2 terms3x³ + 7x² + x - 5 ⇒ 4 terms

How to determine the number of terms in the algebraic expressions

From the question, we have the following parameters that can be used in our computation:

6y + 14 and 3x³ + 7x² + x - 5

Consider an expression given as

ax + b

The number of terms in the expression is 2 and the terms are ax and b

Using the above as a guide, we have the following:

6y + 14 ⇒ 2 terms3x³ + 7x² + x - 5 ⇒ 4 terms

Read more about expression at

https://brainly.com/question/15775046

#SPJ1

Complete question

Identify the number of terms and then the resend themselves for each algebraic expression

6y + 14 and 3x³ + 7x² + x - 5

The area of a circle is 64π cm2, and the area of a sector of the circle is 32π cm2. What is the degree measure of that sector? (Just type the number of degrees. )

Answers

The degree measure of the sector is θ = 180°

We know that the area of a sector is nothing but the area inside the section of the circle created by two radii and an arc.

The formula for the area of a sector of the circle is:

area of a sector = [tex]\frac{\theta}{360} \times \pi \times r^2[/tex]

where θ is the central angle measure in degrees

And πr² is the area of the circle

Here, the area of a sector of the circle A = 32π cm²

and the area of a circle (πr²) =  64π cm²

Using above formula of the area of a sector of the circle,

[tex]A=\frac{\theta}{360} \times \pi \times r^2\\\\32\pi=\frac{\theta}{360} \times \pi \times r^2\\\\32\pi=\frac{\theta}{360} \times 64\times \pi\\\\\theta=\frac{360}{2}\\\\ \theta=180^{\circ}[/tex]

Therefore, the central angle associated to that sector is 180°

Learn more about the area of the circle here:

https://brainly.com/question/28642423

#SPJ4

In right triangle ABC, ZC is a right angle, m ZA = 52, AC = 10. 01, and AB = c.

sin 52° 0. 788

cos 52° 0. 616

tan 52° 1. 280

What is the measurement of AB?

If necessary, round your answer to two decimal places, like this: 42. 53

Answers

The length of side AB rounded to two decimal places is 16.25.

In a right angle triangle, the side on the base of the angle in question is divided by the hypotenuse. In this case, we can use the cosine ratio to find the length of AB.

We are provided with the following information,

C is the right angle, C= 90°

A= 52°

AC = 10. 01

sin 52° = 0. 788

cos 52° = 0. 616

tan 52° = 1. 280

Now,

since angle C is the right angle, AB is the hypotenuse.

Therefore, with respect to Angle A, AC will be the base and BC will be the perpendicular.

We know, cos x= base/hypotenuse

In this case, cos A = AC / AB

cos 52° = 10. 01/ AB

AB = 10. 01 / 0. 616

AB = 16.25

Read more about Trigonometric Ratios:

https://brainly.com/question/25122832

#SPJ4

A line includes the points (0, -8) and (3, 3).
Find the rate of change of the line. (1 point)
uy =

Answers

When the line passes through two points as shown, the slope is 11/3.

What is slope?

The slope or gradient of a line in mathematics is a number that describes both the direction and the steepness of the line. The slope of a line indicates its steepness. Slope is calculated mathematically as "rise over run" (change in y divided by change in x). The slope is a numerical value that describes the steepness of a line and is typically calculated by dividing the vertical distance by the horizontal distance (rise over run) between two points.

Here,

m=3

m=(y2-y1)/(x2-x1)

3=(y+5)/(-3+(2))

-3=y+5

y=-8

The value of slope is 11/3 when the line is passing through 2 points as given.

To know more about slope,

brainly.com/question/29184253

#SPJ1

What are the table of values for y=x^2+2x-3

Answers

By factoring the preceding quadratic equation and then inserting the values of "x," which are -3, -2, -1, 0, 1, 2, 3, 4, 5, the values of "y," which are 12, 5, 0, -3, -4, 3, 0, 5, 12, may be found.

How to find the Calculation?

First, factorize the given quadratic equation before completing the table.

Now, the values of "x" according to the given table are:

x = -3, -2, -1, 0, 1, 2, 3, 4, 5

Add the expression x=-3 to the equation (1).

y = (-3-3)(-3+1)

y = 12

Add the expression x=-2 to the equation (1).

y = (-2-3)(-2+1)

y = 5

Put x=-1 as a value in the equation (1).

y = (-1-3)(-1+1)

y = 0

Put x=0 as the value in the equation (1).

y = (0-3)(0+1)

y = -3

Put x=1 as the value in the equation (1).

y = (1-3)(1+1)

y = -4

Put x=2 as a value in the equation (1).

y = (2-3)(2+1)

y = -3

Add the expression x=3 to the equation (1).

y = (3-3)(3+1)

y = 0

Add the expression x=4 to the equation (1).

y = (4-3)(4+1)

y = 5

Add the expression x=5 to the equation (1).

y = (5-3)(5+1) y = 12

To Learn more About quadratic equation refer To:

https://brainly.com/question/1214333

#SPJ1

For integers a, b, and c, define a*b*c to mean ab-bc+ca then 1*-1*2 equals

Options;
A; -4
B; -2
C; 0
D; 2
E; 4

Answers

The operation of 1 * (-1) * 2 equals option D. 2.

What are Binary Operations?

Binary operations are the operations which are performed on a specific set.

If the operation is defined on a set X, then binary operation can be defined as,

* : X * X → X.

Addition, subtraction, ... are some of the binary operations.

The given binary operation is defined as,

a * b * c = [tex]a^b-b^c+c^a[/tex]

We have to find the value of 1 * (-1) * 2.

By the definition,

1 * (-1) * 2 = [tex]1^{-1}-(-1)^{2}+2^1[/tex]

              = 1 - 1 + 2

              = 2

Hence the correct option is D.

To learn more about Binary Operations, click on the link :

https://brainly.com/question/14058506

#SPJ1

What is the distance between AB and CD?

Answers

√5 is the minimum distance between the line AB and CD.

What are coordinates?

A pair of numbers called coordinates are used to locate a point or a form in a two-dimensional plane. The x-coordinate and the y-coordinate are two numbers that define a point's location on a 2D plane.

Given Two lines AB and CD,

Since, The minimum distance between two lines is the distance between B and C As we can see in the graph

Since, coordinates of B and C are, (1, 2) and (3, 1)

From the formula of distance between two coordinates:

Distance = √((x₂ -x₁)² + (y₂ -y₁)²)

in our case,

Distance = √((3 - 1)²  + (1-2)²)

Distance = √4 + 1)

Distance = √5

Therefore, the minimum distance between the line AB and CD is √5.

Learn more about coordinates here:

https://brainly.com/question/20282507

#SPJ1

2. Amy has a room that is 6 feet by 10 feet. She bought tiles
that are 8 inches by 6 inches. How many tiles will she
need to cover the floor?

Answers

What are units?

A unit can be used for measurement, and is commonly found in mathematics to describe length, size, etc.

Because we are dealing with different units here, we first would need to know how many inches are in one foot.

1 foot = 12 inches

Now that we know how many inches are in a foot, we can convert the length and width of the room from feet to inches.

6 feet = 72 inches

10 feet = 120 inches

Taking these numbers, we now need to figure out the area of the floor.

72 × 120 = 8640

Now, find the area of each tile.

6 × 8 = 48

Taking 8640, we can now divide that by 48 to get the number of tiles needed.

8640 ÷ 48 = 180

Therefore, 180 tiles are needed to cover the floor.

the sum of the cube of a number and the product of the number and twelve is equal to seven times the square of the number. find the number.

Answers

Answer:

4 or 3

Step-by-step explanation:

X³+12X=7X²

X³-7X²+12X=0

X(X²-7X+12)=0

SOLVING THE QUADRATIC EQUATION

you have X=3 or X=4

Nikki gathered data about the length of time she spent listening to the radio and the number of commercials she heard. She organized the data in a scatter plot, where x represents the minutes spent listening and y represents the number of commercials. Then she used the graphing tool to find the equation of the line of best fit:

HELP!!

Answers

By using the equation of the line of best fit, the​ number of minutes Nikki would need to listen to the radio is equal to 63.275 minutes.

How to determine the​ number of minutes Nikki would need to listen to the radio?

In order to determine the​ number of minutes Nikki would need to listen to the radio, we would have to substitute the value representing the number of commercials (y) into the linear equation and then evaluate as follows;

y = 0.338x − 1.387

When the value of y = 20 commercials, the number of minutes (x) in feet is given by;

20 = 0.338x − 1.387

By rearranging and collecting like terms, we have the following:

0.338x = 20 + 1.387

0.338x = 21.387

Number of minutes (x) = 21.387/0.338

Number of minutes (x) = 63.275 minutes.

Read more on line of best fit here: https://brainly.com/question/28165180

#SPJ1

Complete Question:

Nikki gathered data about the length of time she spent listening to the radio and the number of commercials she heard. She organized the data in a scatter plot, where x represents the minutes spent listening and y represents the number of commercials. Then she used the graphing tool to find the equation of the line of best fit: y = 0.338x − 1.387. Based on the line of best fit, for approximately how many minutes will Nikki need to listen to the radio to hear 20 commercials?

A right triangular prism has a triangular base with legs of 5 centimeters and 12 centimeters and a hypotenuse of 13 centimeters. What is the surface area in square centimeters if the height is 2 centimeters?
60
120
180
240
*answer is 180. How?

Answers

The surface area of the triangular prism is 120[tex]cm^2[/tex]

Now, According to the question:

A triangular prism is a polyhedron with a triangle for a base, thus it is a three-sided prism. Given that the triangle has three sides and a height of h, the surface area would be A = Ph + ab where P is the perimeter of the triangle.

We have :

A right triangular prism has a triangular base with legs of 5 centimeters and 12 centimeters and a hypotenuse of 13 centimeters.

Now,

Hypotenuse  = 13

a = 5 cm

b = 12 cm

h = 2 cm

The perimeter of the triangle is the sum of all the sides or:

P = 13 + 5 + 12

P = 30 cm

To get the surface area, substitute the values in the formula below:

A = Ph + ab

A = (30)(2) + (5)(12)

A = 60 + 60

A = 120

Hence, The surface area of the triangular prism is 120[tex]cm^2[/tex]

Learn more about Surface area of triangular at:

https://brainly.com/question/9764079

#SPJ4

A student went on a two-day hike. • On day one, the student hiked 11 kilometers. On day two, the student hiked at a rate of 2 kilometers per hour for x hours. Which of the following graphs represents y, the total distance, in kilometers, the student hiked over both days after hiking for x hours on day two?​

Answers

The slope is 2, which equals the distance travelled each hour. The total distance hiked at the start of the second day is represented by the y-intercept, which is 4, which is also the y-value. intercept's

Explain about the Slope?

We locate the rise/run before calculating the slope. Since the line increases by 2 between each point, the rise is 2. It travels over 1 between the points, hence the run is 1. The slope is now 2/1, or 2.

Given that it is rise/run, this gives us miles/hours, which indicates the number of miles she treks per hour.

The line's crossing of the y-axis is known as the y-intercept. At four, this is.

The fact that the y-value at this time is 4 and the x-value at this point is 0 tells us that she has trekked 4 miles in the 0 hours since the start of the second day, which is the distance she has covered so far.

To learn more about  Slope refer to:

https://brainly.com/question/3493733

#SPJ1

Mock June 22 1F Overview Question Progress Exam Paper Progress 42/80 Marks 10 Eric throws a biased coin 10 times. He gets 3 Tails. Sue throws the same coin 50 times. She gets 20 Tails. 10 Grade For This Paper U 3 Aadi is going to throw the coin once. (i) Which one of the following statements is correct about the probability of Aadi getting Tails? 2/5 (ii) Use Eric's and Sue's results to work out an estimate for the probability that Aadi will get Tails. Write your fraction in the form a/b 13 A Sue's estimate is best because she throws it 50 times. B Sue's estimate is best because she gets more Tails. C Sue's estimate is best because she throws it more times than Eric (1) (1) 15 Total marks: 2 16 17​

Answers

The best estimate for the probability of getting tails is given as follows:

C Sue's estimate is best because she throws it more times than Eric.

How to obtain the probabilities?

A probability is obtained as the division of the number of desired outcomes by the number of total outcomes.

For this problem we calculate an experimental probability, as the outcomes are obtained from the previous trials of Eric and Sue.

The higher the number of trials, the closer the experimental probability is to the theoretical probability, meaning that the correct option is given by option C.

More can be learned about probabilities at https://brainly.com/question/27899440

#SPJ1

Jill and Jessica are bike riding at the park. Jill rides her bike around the two edges of the rectangular park while Jessica takes the short cut. How much farther does Jill ride than Jessica? Round to the nearest tenth.

Answers

Jill ride 43.39 meters farther than Jessica.

What is the Pythagorean theorem?

In a right-angled triangle, the square of the hypotenuse side is equal to the sum of the squares of the other two sides, according to Pythagoras's Theorem. The theorem can be used to determine how steep mountains or slopes are. To calculate the distance between an observer and a location on the ground when the observer is looking down from a tower or structure. It is mostly utilized in the construction industry.

Given, Jill and Jessica are bike riding at the park. Jill rides her bike around the two edges of the rectangular park while Jessica takes the shortcut

Given the length of the Rectangular park = 100m

The width of the park =  60m

Thus jill ride total = 60 + 100

Jill ride total = 160m

Jessica took a shortcut since she rides diagonally.

From the Pythagorean theorem:

hypotaneus² = base² + height²

Thus,

total distance Jessica covers = √(60² + 100²)

total distance Jessica covers = √3600 + 10000

total distance Jessica covers = √13600

total distance Jessica covers = 116.61

Difference between their distances = 160 - 116.61

Difference between their distances = 43.39

therefore, 43.39 m farther does Jill ride than Jessica.

Learn more about the Pythagorean theorem here:

https://brainly.com/question/10174253

#SPJ1

Complete question:

Joshua goes to a store an buys an item that costs x dollars. He has a coupon for 5% off, and then a 6% tax is added to the discounted price. Write an expression in terms of x that represents the total amount that Joshua paid at the register

Answers

The expression representing the total amount paid by Joshua at the register is 1.007x dollars.

We have,

To represent the total amount that Joshua paid at the register, we need to account for the following steps:

- Applying the 5% discount to the original price x:

Discount = 5% of x = 0.05x

Calculating the price after the discount:

Price after discount = x - Discount = x - 0.05x = 0.95 x

Adding the 6% tax to the discounted price:

Tax = 6% of the discounted price = 0.06 x (0.95x)

The expression for the total amount that Joshua paid at the register:

Total amount paid

= Price after discount + Tax

= 0.95 x + 0.06 (0.95 * x)

Simplifying the expression:

Total amount paid = 0.95x + 0.057x

Total amount paid = 1.007x

Thus,

The expression representing the total amount paid by Joshua at the register is 1.007x dollars.

Learn more about expressions here:

https://brainly.com/question/3118662

#SPJ12

The function W
gives the temperature, in degrees Fahrenheit, of a pot of water on a stove t
minutes after the stove is turned on.

After 30 minutes, the pot is taken off the stove.

The graph of the function is shown.

Describe the range of the function.

Answers

The range would be; [75,212] and the greatest one is around 212.5. .5] a domain and a range.

What is the domain and range of the function?

The domain of a function is defined as the set of all the possible input values that are valid for the given function.

The range of a function is defined as the set of all the possible output values that are valid for the given function.

Recall this by saying "DoLaR RiBiT," where "DoLaR" stands for "domain," "L" and "R" stand for "left to right," "R" stands for "range," and "B" and "T" stand for "bottom to top," and "R" stands for "range."

Since we can see that the function doesn't approach 250,  in the graph, 250 is not inside the range.

Between the minimum and greatest value are the numbers that make up the range. The range would be; [75,212] because the lowest Y value is 75 and the greatest one is around 212.5. .5] a domain and a range.

The collection of values that can be substitute into a function's domain are called its parameters.

To learn more about domain refer to :

brainly.com/question/2264373

#SPJ1

HELP ASAP !! :80 POINTS
How long is the shortest route from Ellport to Union Falls using the roads shown?

If necessary, round your answer to the nearest tenth.

Answers

The shortest route from Ellport to Union Falls using road is 18 miles

What is an equation?

An equation is an expression that shows the relationship between numbers and variables.

Pythagoras theorem is used to determine the measure of the sides of a right angled triangle. It is given by:

(hypotenuse side)² = (adjacent side)² + (opposite side)²

The distance from Larksville to Smithfield = 9 + 9 + 6 = 24 miles

The distance from Larksville to Union Falls = 19.5 + 6.5 = 26 miles

Let x represent the distance from Smithfield to Union falls, using Pythaoras:

26² = 24² + x²

x² = 100

x =  10 miles

the distance from Smithfield to Union falls is 10 miles.

Let y be the shortest route from Ellport to Union Falls, using Pythagoras:

y² = 10² + (9 + 6)²

y² = 10² + 15²

y² = 325

y = 18 miles

The distance from Ellport to Union Falls is 18 miles

Find out more on equations at: https://brainly.com/question/2972832

#SPJ1

Other Questions
Can someone help me find the mass of waterHeat absorbed by waterEnergy released by food (calories)Energy released by food (cal/g)% error How are 3 economic questions of whom for, how, and what to produce answered in a command economy? what impact would an increase in net income have on a firm's retained earnings if the firm did not pay a cash dividend? which of the following statements about capitalizing costs is correct? group of answer choices capitalizing costs refers to the process of converting assets to expenses. only the purchase price of the asset is capitalized. capitalizing a cost means to record it as an asset, not as an expense. capitalizing costs results in an immediate decrease in net income. a 70-year-old man who enjoys good health began taking low-dose aspirin several months ago based on recommendations that he read in a magazine article. during the man's most recent visit to his care provider, routine blood work was ordered and the results indicated an unprecedented rise in the man's serum creatinine and blood urea nitrogen (bun) levels. how should a nurse best interpret these findings? Carrying the action potentials, the ____ ____ network spreads throughout the ventricular myocardium to excite the ventricles. why do some store offer senior citizen discounts on tuesdays? group of answer choices senior citizens have perfectly inelastic demand curves, while other shoppers do not. senior citizens have more elastic demands compared to other shoppers. most stores are perfect competitors in their geographic region. senior citizens have less elastic demands compared to other shoppers. determine whether each description best refers to a sole proprietorship, partnership, corporation, or limited liability company (llc). The resistances of then randomly selected 100-ohm resistors have been recorded: 79. 9, 85. 9, 87. 0, 96. 7, 107. 4, 100. 8, 115. 4, 117. 9, 98, and 102. 6 ohms. The individual resistances are known to follow a normal distribution with a mean of 100 ohms and a standard deviation of 10 ohms. (a) Calculate the sample mean resistance associated with the ten resistors. (b) For a sample size of ten, find the probability of obtaining a sample mean as large as or larger than the one calculated in part (a). (c) For a sample size of ten, find the probability of obtaining a sample mean with a resistance between 95 and 105 ohms Souligner le discours indirect.Et le lendemain matin, Bob me confirma avec la mme simplicit, avec ce mme sens de lhospitalit immdiate, que seul possde les Amricains, que jtais linvit de la maison Kendall pour quinze jours Dallas, Texas, o il est n. Experimental data indicate that in a region downstream of a given louvered supply vent the velocity of the emitted air is defined by v=0.18v0/x, where v and x are expressed in m/s and meters, respectively, and v0 is the initial discharge velocity of the air. For v0=3.6m/s, determine:a) the acceleration of the air at x=2m,b) the time required for the air to flow from x=1 to x=3m. Determine whether the pairs of triangles are congruent.~Circle yes, no, or not enough information. Explain your choice. What is the perimeter of the given composite figure? I believe its 140 but would like a check from someone smarter than I lol. This states that charge cannot be created or destroyed. However, a charge can be transferred from one object to other leaders such as thomas jefferson formed committees of correspondence that: established colonial legislatures.began writing a national constitution.urged resistance to the British.sent soldiers to fight in the French and Indian War. the following mechanism had been proposed for the reaction of h2(g) with icl(g). what rate law would be observed if this is the correct mechanism? 24. in problem 23, how many different paths are there from a to b that go through the point circled in the following lattice? b ross, sheldon. first course in probability, a (p. 17). pearson education. kindle edition. What was the King of England's role in relationship to theKing of France? suppose you are going to a film festival. the admission price for the festival is $20. upon entry, you can view films for a cost of $1 each. the marginal cost of the fifth film is __________ true or false rf values should be kept in the range of 0.2 to 0.8 for the most effective separation when more than one compound is present in the sample.